Tricky Trace

Algebra Level 5

Consider an n × n n\times n square matrix A A such that A i j = 1 min ( i , j ) A_{ij}=\dfrac{1}{\min(i,j)} .

Let t n = tr ( A 1 ) t_n=\operatorname{tr}(A^{-1})

Find n = 1 1 t n + 1 \displaystyle \sum_{n=1}^{\infty}\dfrac{1}{\left|t_n\right|+1}

Note: tr ( X ) \operatorname{tr}(X) denotes the trace i.e. the sum of the diagonal entries of the matrix X X .

7 8 \dfrac{7}{8} 3 8 \dfrac{3}{8} 8 9 \dfrac{8}{9} 4 9 \dfrac{4}{9}

This section requires Javascript.
You are seeing this because something didn't load right. We suggest you, (a) try refreshing the page, (b) enabling javascript if it is disabled on your browser and, finally, (c) loading the non-javascript version of this page . We're sorry about the hassle.

1 solution

Mark Hennings
Jun 19, 2020

Let A ( n ) A(n) be the n × n n\times n version of this matrix, and let F n ( t ) = d e t ( t I n A ( n ) ) F_n(t) = \mathrm{det}\,\big(t I_n - A(n)\big) be the characteristic polynomial of the matrix A ( n ) A(n) . Then F 1 ( t ) = t 1 F_1(t) = t-1 and F 2 ( t ) = t 2 3 2 t 1 2 F_2(t) = t^2 - \tfrac32t - \tfrac12 . If n 3 n \ge 3 we can use elementary row operations to show that F n ( t ) = 1 1 1 2 1 2 t I n 2 A ( n 2 ) 1 n 2 1 n 2 1 1 2 1 n 2 t 1 n 1 1 n 1 1 1 2 1 n 2 1 n 1 t 1 n = 1 1 1 2 1 2 t I n 2 A ( n 2 ) 1 n 2 1 n 2 1 1 2 1 n 2 t 1 n 1 1 n 1 0 0 0 t t + 1 n ( n + 1 ) = ( t + 1 n ( n 1 ) ) 1 1 2 t I n 2 A ( n 2 ) 1 n 2 1 1 2 1 n 2 t 1 n 1 + t 1 1 2 t I n 2 A ( n 2 ) 1 n 2 1 1 2 1 n 2 1 n 1 = ( t + 1 n ( n 1 ) ) F n 1 ( t ) + t d e t B n 1 ( t ) \begin{aligned} F_n(t) & = \; \left|\begin{array}{ccccc|cc} & & & & & -1 & -1 \\ & & & & & -\frac12 & -\frac12 \\ & & tI_{n-2} - A(n-2) & & & \vdots & \vdots \\ & & & & & \vdots & \vdots \\ & & & & & -\frac{1}{n-2} & -\frac{1}{n-2} \\ \hline -1 & -\frac12 & \cdots & \cdots & -\frac{1}{n-2} & t - \frac{1}{n-1} & -\frac{1}{n-1} \\ -1 & -\frac12 & \cdots & \cdots & -\frac{1}{n-2} & -\frac{1}{n-1} & t - \frac{1}{n} \end{array} \right| \; = \; \left|\begin{array}{ccccc|cc} & & & & & -1 & -1 \\ & & & & & -\frac12 & -\frac12 \\ & & tI_{n-2} - A(n-2) & & & \vdots & \vdots \\ & & & & & \vdots & \vdots \\ & & & & & -\frac{1}{n-2} & -\frac{1}{n-2} \\ \hline -1 & -\frac12 & \cdots & \cdots & -\frac{1}{n-2} & t - \frac{1}{n-1} & -\frac{1}{n-1} \\ 0 & 0 & \cdots & \cdots & 0 & -t & t + \frac{1}{n(n+1)} \end{array} \right| \\[4ex] & = \; \left(t + \tfrac{1}{n(n-1)}\right)\left|\begin{array}{ccccc|c} & & & & & -1 \\ & & & & & -\frac12 \\ & & tI_{n-2} - A(n-2) & & & \vdots \\ & & & & & \vdots \\ & & & & & -\frac{1}{n-2} \\ \hline -1 & -\frac12 & \cdots & \cdots & -\frac{1}{n-2} & t - \frac{1}{n-1} \end{array} \right| + t\left|\begin{array}{ccccc|c} & & & & & -1 \\ & & & & & -\frac12 \\ & & tI_{n-2} - A(n-2) & & & \vdots \\ & & & & & \vdots \\ & & & & & -\frac{1}{n-2} \\ \hline -1 & -\frac12 & \cdots & \cdots & -\frac{1}{n-2} & -\frac{1}{n-1} \end{array} \right| \\ & = \; \left(t + \tfrac{1}{n(n-1)}\right)F_{n-1}(t) + t\mathrm{det}\,B_{n-1}(t) \end{aligned} where B n 1 ( t ) B_{n-1}(t) is the ( n 1 ) × ( n 1 ) (n-1) \times (n-1) matrix that only differs from t I n 1 A ( n 1 ) tI_{n-1} - A(n-1) by having 1 n 1 -\tfrac{1}{n-1} , instead of t 1 n 1 t - \tfrac{1}{n-1} , as the bottom right element. In particular, then d e t B n 1 ( 0 ) = d e t ( A ( n 1 ) ) = F n 1 ( 0 ) \mathrm{det}\,B_{n-1}(0) = \mathrm{det}(-A(n-1)) = F_{n-1}(0) . We thus obtain the recurrence relations F n ( 0 ) = 1 n ( n 1 ) F n 1 ( 0 ) F n ( 0 ) = 1 n ( n 1 ) F n 1 ( 0 ) + 2 F n 1 ( 0 ) n 3 F_n(0) \; = \; \tfrac{1}{n(n-1)}F_{n-1}(0) \hspace{2cm} F_n'(0) \; = \; \tfrac{1}{n(n-1)}F_{n-1}'(0) + 2F_{n-1}(0) \hspace{2cm} n \ge 3 We can solve these recurrence relations to deduce that n ! ( n 1 ) ! F n ( 0 ) = 1 , n ! ( n 1 ) ! F n ( 0 ) = 1 2 3 ( n 1 ) n ( n + 1 ) n 1 n!(n-1)! F_n(0) \; = \; -1 \hspace{1cm} , \hspace{1cm} n!(n-1)!F_n'(0) \; =\; 1 - \tfrac23(n-1)n(n+1) \hspace{2.5cm} n \ge 1

Since A ( n ) A(n) is a symmetric matrix, it is diagonalisable, and hence, if F n ( t ) = t n + a n 1 t n 1 + a n 2 t n 2 + + a 1 t + a 0 F_n(t) \; = \; t^n + a_{n-1}t^{n-1} + a_{n-2}t^{n-2} + \cdots + a_1t + a_0 then T r A ( n ) \mathrm{Tr}\,A(n) is just the sum of the eigenvalues of A ( n ) A(n) , and hence equal to a n 1 -a_{n-1} . The matrix A ( n ) 1 A(n)^{-1} is also symmetric, and its characteristic polynomial is the reverse of F n ( t ) F_n(t) , namely 1 a 0 ( a 0 t n + a 1 t n 1 + + a n 1 t + a n ) \frac{1}{a_0}\left(a_0t^n + a_1t^{n-1} + \cdots + a_{n-1}t + a_n\right) and hence we deduce that t n = T r A ( n ) 1 = a 1 a 0 = F n ( 0 ) F n ( 0 ) = 1 2 3 ( n 1 ) n ( n + 1 ) n 1 t_n \; = \; \mathrm{Tr}\,A(n)^{-1} \; = \; -\frac{a_1}{a_0} \; = \; -\frac{F_n'(0)}{F_n(0)} \; = \; 1 - \tfrac23(n-1)n(n+1) \hspace{2cm} n \ge 1 and hence, finally, n = 1 1 t n + 1 = 1 2 + n = 2 3 2 ( n 1 ) n ( n + 1 ) = 1 2 + 3 4 n = 2 ( 1 ( n 1 ) n 1 n ( n + 1 ) ) = 1 2 + 3 8 = 7 8 \sum_{n=1}^\infty \frac{1}{|t_n|+1} \; = \; \frac12 + \sum_{n=2}^\infty \frac{3}{2(n-1)n(n+1)} \; = \; \frac12 + \frac34\sum_{n=2}^\infty\left(\frac{1}{(n-1)n} - \frac{1}{n(n+1)}\right) \; = \; \frac12 + \frac38 \; = \; \boxed{\frac78}


An alternative approach to calculating T r [ A ( n ) 1 ] \mathrm{Tr}\big[A(n)^{-1}\big] relies on obtaining an inductive formula for the inverse matrix A ( n ) 1 A(n)^{-1} . Note that ( 0 A ( n ) 1 0 0 0 0 n ( n + 1 ) n ( n + 1 ) ) A ( n + 1 ) = ( 0 A ( n ) 1 0 0 0 0 n ( n + 1 ) n ( n + 1 ) ) ( 1 1 2 A ( n ) 1 n 1 1 2 1 n 1 n + 1 ) = ( 0 0 I n 0 1 0 0 0 0 1 ) \begin{aligned} \left(\begin{array}{ccccc|c} & & & & & 0 \\ & & & & & \vdots \\ & & A(n)^{-1} & & & \vdots \\ & & & & & 0 \\ & & & & & 0 \\\hline 0 & 0 & \cdots & & n(n+1)& - n(n+1) \end{array} \right)A(n+1) & = \; \left(\begin{array}{ccccc|c} & & & & & 0 \\ & & & & & \vdots \\ & & A(n)^{-1} & & & \vdots \\ & & & & & 0 \\ & & & & & 0 \\\hline 0 & 0 & \cdots & & n(n+1)& - n(n+1) \end{array} \right)\left(\begin{array}{ccccc|c} & & & & & 1 \\ & & & & & \frac12 \\ & \; A(n) & & & \vdots \\ & & & & & \vdots \\ & & & & & \frac{1}{n} \\\hline 1 & \frac12 & \cdots & \cdots & \frac{1}{n} & \frac{1}{n+1} \end{array}\right) \\ & = \; \left(\begin{array}{ccccc|c} & & & & & 0 \\ & & & & & 0 \\ & & I_n & & & \vdots \\ & & & & & 0 \\ & & & & & 1 \\\hline 0 & 0 & \cdots & 0 & 0 & 1 \end{array}\right) \end{aligned} so that A ( n + 1 ) 1 = ( 0 0 I n 0 1 0 0 0 1 ) ( 0 A ( n ) 1 0 0 0 0 0 n ( n + 1 ) n ( n + 1 ) ) = ( 0 0 A ( n ) 1 n ( n + 1 ) K ( n ) 0 n ( n + 1 ) 0 0 0 n ( n + 1 ) n ( n + 1 ) ) \begin{aligned} A(n+1)^{-1} & = \; \left(\begin{array}{ccccc|c} & & & & & 0 \\ & & & & & 0 \\ & & I_n & & & \vdots \\ & & & & & 0 \\ & & & & & -1 \\\hline 0 & 0 & \cdots & \cdots & 0 & 1 \end{array}\right)\left(\begin{array}{ccccc|c} & & & & & 0 \\ & & & & & \vdots \\ & & A(n)^{-1} & & & \vdots \\ & & & & & 0 \\ & & & & & 0 \\\hline 0 & 0 & \cdots & 0 & n(n+1)& - n(n+1) \end{array} \right) \\ & = \; \left(\begin{array}{ccccc|c} & & & & & 0 \\ & & & & & 0 \\ & & A(n)^{-1} - n(n+1)K(n) & & & \vdots \\ & & & & & 0 \\ & & & & & n(n+1) \\\hline 0 & 0 & \cdots & 0 & n(n+1) & -n(n+1) \end{array}\right) \end{aligned} where K ( n ) K(n) is the n × n n \times n matrix with zeros in all components except the bottom right one, which has the value 1 1 . From this it seems that A ( n ) 1 A(n)^{-1} is always a tridiagonal matrix, but we do not need to identify it orecisely. We see that T r [ A ( n + 1 ) 1 ] = T r [ A ( n ) 1 n ( n + 1 ) K ( n ) ] n ( n + 1 ) = T r [ A ( n ) 1 ] 2 n ( n + 1 ) n 1 \mathrm{Tr}\big[A(n+1)^{-1}\big] \; =\; \mathrm{Tr}\big[A(n)^{-1} - n(n+1)K(n)\big] - n(n+1) \; = \; \mathrm{Tr}\big[A(n)^{-1}\big] - 2n(n+1) \hspace{1cm} n \ge 1 and hence we deduce that T r [ A ( n ) 1 ] = 1 2 r = 1 n 1 r ( r + 1 ) = 1 2 3 ( n 1 ) n ( n + 1 ) n 1 \mathrm{Tr}\big[A(n)^{-1}\big] \; =\; 1 - 2\sum_{r=1}^{n-1}r(r+1) \; = \; 1 - \tfrac23(n-1)n(n+1) \hspace{2cm} n \ge 1

i got it right but forgot to add 1/2 at the end. sad me

Lingga Musroji - 11 months, 3 weeks ago

I was inspired to post this problem after seeing a similar problem that appeared in the Putnam Competition of 2014.

Check out the problem A2 here

Click here for the solution

Digvijay Singh - 11 months, 3 weeks ago

Log in to reply

Indeed. My solution already gives the answer to A2, since it is ( 1 ) n F n ( 0 ) = ( 1 ) n 1 n ! ( n 1 ) ! (-1)^nF_n(0) = \frac{(-1)^{n-1}}{n!(n-1)!} . By the end of "Trickier Trace", I have identified the inverse of A ( n ) A(n) as the symmetric tridiagonal matrix mentioned in passing in the Putnam solution.

Mark Hennings - 11 months, 3 weeks ago

0 pending reports

×

Problem Loading...

Note Loading...

Set Loading...